Đến nội dung

Hoang Tung 126 nội dung

Có 1000 mục bởi Hoang Tung 126 (Tìm giới hạn từ 09-06-2020)



Sắp theo                Sắp xếp  

#466674 Tìm x,y thuộc $N^{*}$ thoả mãn $$\left...

Đã gửi bởi Hoang Tung 126 on 25-11-2013 - 15:01 trong Đại số

Ta có :$x=\frac{y+1}{l}= > ky-1=\frac{y+1}{l}= > kly-l=y+1= > y(kl-1)=l+1= > y=\frac{l+1}{kl-1}$

Do y là số tự nhiên $= > l+1\vdots kl-1= > l+1\geq kl-1= > l\geqslant kl= > l(k-1)\leq 0= > k-1\leq 0= > k\leq 1$

Do k nguyên dương nên k=1 $= > y=\frac{l+1}{l-1}$

$= > l+1\vdots l-1= > l-1+2\vdots l-1= > 2\vdots l-1$ $= > l-1=1,l-1=2= > l=2$ hoặc $l=3$

-Với $k=1,l=2= > 2x-1=y,y-1=x= > y=3,x=2$

-Voi $k=1,l=3= > 3x-1=y,y-1=x= > y=2,x=1$




#477087 Tứ Giác Đều Hòa

Đã gửi bởi Hoang Tung 126 on 13-01-2014 - 18:54 trong Hình học

Sử dụng tam giác đồng dạng ta CM được :$AB.CD=AC.BD$ nên ABCD điều hoà




#482659 Tìm GTLN, GTNN của $y=x^{3}-6x^{2}+21x+18$

Đã gửi bởi Hoang Tung 126 on 12-02-2014 - 09:26 trong Bất đẳng thức và cực trị

Cho $-\frac{1}{2}\leq x\leq 1$. Tìm GTLN, GTNN của $y=x^{3}-6x^{2}+21x+18$

Ta có:$y=x^3-6x^2+12x+18=x^2(x-1)-5x(x-1)+7(x-1)+25=(x-1)(x^2-5x+7)+25\leq 25$(Do $x-1\leq 0,x^2-5x+7> 0$

$4y=4x^3-24x^2+84x+72=2x^2(2x+1)-13x(2x+1)+\frac{97}{2}(2x+1)+72-\frac{97}{2}=(2x+1)(2x^2-13x+\frac{97}{2})+\frac{47}{2}\geq \frac{47}{2}= > y\geq \frac{47}{8}$




#469853 Tìm GTLN của biểu thức với x,y,z>0;x+y+z=1

Đã gửi bởi Hoang Tung 126 on 09-12-2013 - 14:27 trong Bất đẳng thức và cực trị

Theo bdt AM-GM có :$(a+b)(b+c)(c+a)\leq\frac{8(a+b+c)^3}{27}=\frac{8}{27}$

                                 $abc\leq \frac{(a+b+c)^3}{27}=\frac{1}{27}$

$= > abc(a+b)(b+c)(c+a)\leq \frac{8}{27}.\frac{1}{27}=\frac{8}{27^2}$

Dang thức xảy ra khi $a=b=c=\frac{1}{3}$




#470089 Tìm GTLN của biểu thức với x,y,z>0;x+y+z=1

Đã gửi bởi Hoang Tung 126 on 10-12-2013 - 15:21 trong Bất đẳng thức và cực trị

hiểu rồi à? phần bđt này mình mới học hồi chiều. công nhận mấy cái này khó thật...

Phần này cũng bình thường mà chỉ cẩn áp dụng nhớ là nó phải dương




#469861 Tìm GTLN của biểu thức với x,y,z>0;x+y+z=1

Đã gửi bởi Hoang Tung 126 on 09-12-2013 - 15:13 trong Bất đẳng thức và cực trị

Bạn ơi biến x,y,z mà???

Chắc mình ghi nhầm




#470076 Tìm các số nguyên a,b,c để hệ phương trình có nghiệm nguyên:

Đã gửi bởi Hoang Tung 126 on 10-12-2013 - 15:00 trong Số học

Cộng theo vế các pt ta được $x^2(a+b+c)+x(a+b+c)+a+b+c=0= > (a+b+c)(x^2+x+1)=0= > a+b+c=0= > c=-a-b$

Thay vào pt đầu ta được $ax^2+bx-a-b=0< = > a(x-1)(x+1)+b(x-1)=0< = > (x-1)(ax+a+b)=0= > ax+b=-a= > cx^2-a=0= > x^2=\frac{a}{c}= > a\vdots c$

Tương tự $c\vdots a$

$= > c=a$.Thay vào (1) ta được :$ax^2+bx+a=0$




#560170 $\sum \sqrt[3]{\frac{a^2+bc}{abc(b^2+...

Đã gửi bởi Hoang Tung 126 on 18-05-2015 - 16:02 trong Bất đẳng thức và cực trị

 Cho các số thực dương $a,b,c$. CMR:

 

  $\sum \sqrt[3]{\frac{a^2+bc}{abc(b^2+c^2)}}\geq \frac{9}{a+b+c}$




#560173 $\sum \sqrt[3]{\frac{a^2+bc}{abc(b^2+...

Đã gửi bởi Hoang Tung 126 on 18-05-2015 - 16:07 trong Bất đẳng thức và cực trị

Đề nghị mod cho sao chép bài này




#560171 $\sum \sqrt[3]{\frac{a^2+bc}{abc(b^2+...

Đã gửi bởi Hoang Tung 126 on 18-05-2015 - 16:06 trong Bất đẳng thức và cực trị

 Theo AM-GM 3 số ta có :

 

  $\sum \sqrt[3]{\frac{a^2+bc}{abc(b^2+c^2)}}=\sum \frac{a^2+bc}{\sqrt[3]{abc(b^2+c^2)(a^2+bc)^2}}=\sum \frac{a^2+bc}{\sqrt[3]{(ab^2+ac^2)(ba^2+bc^2)(ca^2+c^2b)}}\geq \sum \frac{a^2+bc}{\frac{ab^2+ac^2+ba^2+bc^2+ca^2+c^2b}{3}}=3\sum \frac{a^2+bc}{\sum ab(a+b)}=\frac{3\sum a^2+3\sum ab}{\sum ab(a+b)}$

 

Ta cần chứng minh $\frac{3\sum a^2+3\sum ab}{\sum ab(a+b)}\geq \frac{9}{\sum a}< = > (\sum a^2+\sum ab)(\sum a)\geq 3\sum ab(a+b)< = > \sum a^3+3abc\geq \sum ab(a+b)< = > a(a-b)(a-c)+b(b-c)(b-a)+c(c-a)(c-b)\geq 0$

 

 BDT này luôn đúng vì đó là Schur bậc 3.

 

    Dấu = xảy ra khi $a=b=c$




#475052 Đề thi HSG huyện Yên Thành VÒNG 2

Đã gửi bởi Hoang Tung 126 on 03-01-2014 - 20:15 trong Tài liệu - Đề thi

Bài 3: (Bài ngon nhất ) :Không mất tính tổng quát giả sử c là số lớn nhất trong 3 số $= > 1\leq c\leq 2$

Ta có :$a^2+b^2+c^2=(a+b)^2+c^2-2ab\leq (a+b)^2+c^2=(3-c)^2+c^2\leq 5< = > 2c^2-6c+9\leq 5< = > c^2-3c+2\leq 0< = > (c-1)(c-2)\leq 0$




#468019 Chứng minh: $a+b=c$ thì $a^{4}+b^{4}+c^...

Đã gửi bởi Hoang Tung 126 on 01-12-2013 - 07:35 trong Đại số

Bài 3:Quy đồng lên BĐT $< = > a^2b^2+b^2c^2+c^2a^2\geq abc(a+b+c)$

Theo bđt Cosi có :$a^2b^2+b^2c^2\geq 2\sqrt{a^2b^4c^2}=2ab^2c$

Thiết lập các bddt tương tự rồi cộng lại ta có đpcm




#468018 Chứng minh: $a+b=c$ thì $a^{4}+b^{4}+c^...

Đã gửi bởi Hoang Tung 126 on 01-12-2013 - 07:33 trong Đại số

Bài 2: BĐT $< = > \frac{1}{2}(a-b)^2+\frac{1}{2}(b-c)^2+\frac{1}{2}(c-a)^2\geq 0$(luôn đúng)




#467329 Tìm các giá trị $m$ để phương trình có 2 nghiệm thực

Đã gửi bởi Hoang Tung 126 on 28-11-2013 - 18:13 trong Phương trình, hệ phương trình và bất phương trình

à mình quên để mình sửa lại.Tks

Không có gì




#467318 Tìm các giá trị $m$ để phương trình có 2 nghiệm thực

Đã gửi bởi Hoang Tung 126 on 28-11-2013 - 17:37 trong Phương trình, hệ phương trình và bất phương trình

Bạn thiếu điều kiện $x\geq 1$

 

PT <=> 2x2-2(m + 2)x - 2m +7 = x2 - 2x +1

     <=> x2 - 2(m +1)x - (2m - 7) = 0

          $\Delta$' = m2 + 8 > 0

=> PT có 2 nghiêm thực

:namtay  :namtay  :namtay  :namtay  :namtay  :namtay  :namtay  :icon11:  :icon11:  :icon11:  :icon11: 




#460211 Cho tam giác ABC có O là điểm nằm trong tam giác

Đã gửi bởi Hoang Tung 126 on 27-10-2013 - 09:08 trong Hình học phẳng

 Cho $\Delta ABC$ có $O$ là điểm bất kỳ nằm trong $\Delta$. Nối $AO,BO,CO$ giao $BC,CA,AB$ ở $M,N,P$. Gọi $I$ là điểm bất kỳ nằm trong $\Delta MNP$ .Nối $MI,NI,PI$ giao $PN,PM,MN$ ở $D,E,F$. CMR:  $AD,BE,CF$ đồng quy.




#561069 $\frac{3(\sum ab)}{(\sum a)^{2}...

Đã gửi bởi Hoang Tung 126 on 23-05-2015 - 09:05 trong Bất đẳng thức - Cực trị

Cho $a,b,c>0$ . CMR : $\frac{3(\sum ab)}{(\sum a)^{2}}+\frac{\prod (a+b)}{8abc}\geq 2$

BDT $< = > \frac{\prod (a+b)}{8abc}-1\geq 1-\frac{3\sum ab}{(\sum a)^2}< = > \frac{\sum ab(a+b)-6abc}{8abc}\geq \frac{\sum a^2-\sum ab}{(\sum a)^2}< = > \frac{\sum a(b-c)^2}{4abc}\geq \frac{\sum (b-c)^2}{(\sum a)^2}< = > \sum (b-c)^2(\frac{1}{4bc}-\frac{1}{(\sum a)^2})\geq 0$

 

  BDT này đúng do $4bc\leq (b+c)^2< (a+b+c)^2= > \frac{1}{4bc}> \frac{1}{(\sum a)^2}= > \frac{1}{4bc}-\frac{1}{(\sum a)^2}> 0$

 

Tương tự $\frac{1}{4ac}-\frac{1}{(\sum a)^2}> 0,\frac{1}{4ab}-\frac{1}{(\sum a)^2}> 0$

 

 Do đó $S_{a}> 0,S_{b}> 0,S_{c}> 0$ nên ta có ĐPCM




#453216 Cho a,b,c>0 và a+b+c=1. CMR: $\sum \frac{a^2+b}...

Đã gửi bởi Hoang Tung 126 on 26-09-2013 - 20:16 trong Bất đẳng thức và cực trị

Lưu ý là có đk :a+b+c=1 rồi thay vào thôi




#453379 chứng minh n chẵn

Đã gửi bởi Hoang Tung 126 on 27-09-2013 - 16:16 trong Số học

Ta có :$3^n-1=(3-1)(3^(n-1)+3^(n-2)+...+1)=2(3^(n-1)+3^(n-2)+...+1)$ là số chẵn nên n chẵn




#453112 Cho a,b,c>0 và a+b+c=1. CMR: $\sum \frac{a^2+b}...

Đã gửi bởi Hoang Tung 126 on 26-09-2013 - 14:15 trong Bất đẳng thức và cực trị

Ta có :$\frac{a^2+b}{b+c}+a+\frac{b^2+c}{c+a}+b+\frac{c^2+a}{a+b}+c=\frac{a(a+b+c)+b}{b+c}+\frac{b(b+c+a)+c}{c+a}+\frac{c(c+a+b)+a}{a+b}=\frac{a(a+b+c)+b(a+b+c)}{b+c}+\frac{b(a+b+c)+c(a+b+c)}{c+a}+\frac{c(c+a+b)+a(a+b+c)}{a+b}=\frac{(a+b+c)(a+b)}{b+c}+\frac{(a+b+c)(b+c)}{c+a}+\frac{(c+a)(a+b+c)}{b+a}=(a+b+c)(\frac{a+b}{b+c}+\frac{b+c}{c+a}+\frac{c+a}{a+b})\geq 3(a+b+c)\sqrt[3]{\frac{(a+b)(b+c)(c+a)}{(a+b)(b+c)(c+a)}}=3(a+b+c)= > \frac{a^2+b}{b+c}+\frac{b^2+c}{c+a}+\frac{c^2+a}{a+b}\geq 3(a+b+c)-(a+b+c)=2(a+b+c)=2$(đpcm) 

 Dấu =xảy ra khi a=b=c=$\frac{1}{3}$




#476319 M=$\sum \frac{\sqrt{x^2+xy+y^2}}...

Đã gửi bởi Hoang Tung 126 on 09-01-2014 - 16:55 trong Bất đẳng thức - Cực trị

Ta có :$M=\sum \frac{\sqrt{x^2+xy+y^2}}{4yz+1}\geq \sum \frac{\sqrt{\frac{3}{4}(x+y)^2}}{4yz+1}=\frac{\sqrt{3}}{2}.\sum \frac{x+y}{4yz+1}\geq \frac{\sqrt{3}}{2}.\sum \frac{x+y}{(y+z)^2+1}$$M=\sum \frac{\sqrt{x^2+xy+y^2}}{4yz+1}\geq \sum \frac{\sqrt{\frac{3}{4}(x+y)^2}}{4yz+1}=\frac{\sqrt{3}}{2}.\sum \frac{x+y}{4yz+1}\geq \frac{\sqrt{3}}{2}.\sum \frac{x+y}{(y+z)^2+1}$

Đặt $y+z=a,x+z=b,x+y=c$ $= > a+b+c=2(x+y+z)=3$

$= > A\geq\frac{\sqrt{3}}{2}. \sum \frac{c}{a^2+1}=\frac{\sqrt{3}}{2}.\sum \frac{c(a^2+1)-a^2c}{a^2+1}=\frac{\sqrt{3}}{2}.(\sum c-\sum \frac{a^2c}{a^2+1})\geq \frac{\sqrt{3}}{2}.(3-\sum \frac{a^2c}{2a})=\frac{\sqrt{3}}{2}(3-\sum \frac{ac}{2})\geq \frac{\sqrt{3}}{2}(3-\frac{(a+b+c)^2}{6})=\frac{\sqrt{3}}{2}(3-\frac{9}{6})=\frac{3\sqrt{3}}{4}$

 Dấu=  xảy ra tại a=b=c=1 hay $x=y=z=\frac{1}{2}$




#483825 $\frac{1}{1+xy}+\frac{1}{1+...

Đã gửi bởi Hoang Tung 126 on 18-02-2014 - 14:21 trong Bất đẳng thức - Cực trị

Cho $x,y,z\in \left ( 0,1 \right ]$ .CMR

                               $\frac{1}{1+xy}+\frac{1}{1+yz}+\frac{1}{1+zx}\leq \frac{5}{x+y+z}$

Ta có:$(1-x)(1-y)\geq 0= > xy+1\geq x+y= > \frac{1}{xy+1}\leq \frac{1}{x+y}$.Tương tự $\frac{1}{yz+1}\leq \frac{1}{y+z},\frac{1}{xz+1}\leq \frac{1}{x+z}$

Cộng theo vế $= > \sum \frac{1}{xy+1}\leq \sum \frac{1}{x+y}$

Do đó ta cần CM:$\frac{1}{x+y}+\frac{1}{y+z}+\frac{1}{x+z}\leq \frac{5}{x+y+z}< = > \left [ (x+y)+(x+z)+(y+z) \right ](\frac{1}{x+y}+\frac{1}{y+z}+\frac{1}{x+z})\leq 10$

Đặt $x+y=a,y+z=b,x+z=c= > 0\leq a,b,c\leq 2$

BDT$< = > (a+b+c)(\frac{1}{a}+\frac{1}{b}+\frac{1}{c})\leq 10$.Đây chính là bdt quen thuộc




#570148 $\sum \frac{a}{a+b+7c}+\frac{2(...

Đã gửi bởi Hoang Tung 126 on 06-07-2015 - 08:20 trong Bất đẳng thức - Cực trị

  Bài toán : Cho các số thực dương $a,b,c> 0$ .CMR:

 

  $\frac{a}{a+b+7c}+\frac{b}{b+c+7a}+\frac{c}{c+a+7b}+\frac{2(ab+bc+ac)}{3(a^2+b^2+c^2)}\leq 1$

 

 




#464468 CMR: $\sqrt{\frac{a}{b}}+\s...

Đã gửi bởi Hoang Tung 126 on 15-11-2013 - 15:38 trong Bất đẳng thức và cực trị

Bài 3: Không mất tính tổng quát giả sử $a\geq b\geq c$

$= > c^2a\geq c^2.c=c^3$

Từ đề bài ta cần CM :$3+a^2b+b^2c\geq c^3+2a^3+2b^3$

Mặt khác do $a\geqb \geq c= > a^2b-b^3=b(a^2-b^2)\geq 0= > a^2b\geq b^3,b^2c\geq c^3$(1)

Mà $0< b< 1= > b^3< 1$(2)

Cộng theo vế (1) và (2) $= > a^2b+b^2c+1> b^3+c^3+b^3=2b^3+c^3$(3)

Do $0< a<1 = > 2a^3< 2$(4)

Cộng theo vế (3) và (4) $= > a^2b+b^2c+1+2> 2b^3+c^3+2a^3$(đpcm)

$= > 2a^3+2b^3+2c^3< 3+a^2b+b^2c+c^2a$




#464466 CMR: $\sqrt{\frac{a}{b}}+\s...

Đã gửi bởi Hoang Tung 126 on 15-11-2013 - 15:26 trong Bất đẳng thức và cực trị

Bài 1: Đặt $\sqrt{a}=x,\sqrt{b}=y$ .BĐT $< = > \frac{x}{y}+\frac{y}{x}+\frac{3(x+y)}{\sqrt{x^2+y^2}}> 6$

Theo bđt AM-GM  có :$\frac{x}{y}+\frac{y}{x}+\frac{3(x+y)}{\sqrt{x^2+y^2}}\geq \frac{x}{y}+\frac{y}{x}+\frac{6\sqrt{xy}}{\sqrt{x^2+y^2}}=\frac{x^2+y^2}{xy}+\frac{6\sqrt{xy}}{\sqrt{x^2+y^2}}=\frac{x^2+y^2}{xy}+\frac{3\sqrt{xy}}{\sqrt{x^2+y^2}}+\frac{3\sqrt{xy}}{\sqrt{x^2+y^2}}\geq 3\sqrt[3]{\frac{9xy(x^2+y^2)}{xy(x^2+y^2)}}=3\sqrt[3]{9}> 3\sqrt[3]{8}=3.2=6$(đpcm)